What is the answer to P(4)=4n + 5

Answers

Answer 1

Answer:

P(x) = 4n+5

P(4) = 4(4)+5

P(4) = 16+5

P(4) = 21

The answer is 21.

Let me know if this helps!


Related Questions

HELPASAP (15 points)
A circle with an arc length of ____ centimeters is intercepted by a central angle of 3pi/4 radians has a radius of ____ centimeters.

1st Blank Options: 12pi, 4pi, 2pi
2nd Blank Options: 3, 16, 24

Answers

The 1st blank is 12pi and the 2nd blank is 16 hope this helps.

If p varies inversely as q and p =8 when q=10,find p when q=4

Answers

Answer:

20

Step-by-step explanation:

When two numbers vary inversely, they always multiply to the same product, in this case, it is 80, because it is 8 x 10(product of two known values of p and q). This means, for q = 4, we can write:

p * 4 = 80

This means that p is equal to 20.

The value of p is 20 when q=4

What is directly proportional and inversely proportional relationship?

Let there are two variables p and q

Then, p and q are said to be directly proportional to each other if

p = kq

where k is some constant number called constant of proportionality.

When two numbers vary inversely, then the will always multiply to the same product,

Here it is 80, because it is 8 x 10

Then product of two known values of p and q).

This means, for q = 4, we can write:

p x 4 = 80

This means that p is 20.

Learn more about directly inversely proportional relationship variable here:

https://brainly.com/question/13082482

#SPJ2

which of the following equations is not a function?

a.) 5y^2 = x

b.) 1/2y + 9 = x^2 + 2x + 1

c.) y = 1/2x + 9

d.) 8x + 4y = 0 ​

Answers

y isnt a function

hope this helps

Instructions: Find the missing side. Round your answer to the nearest tenth.
36
Х
71°

Answers

Answer:

x = 12.4

Step-by-step explanation:

Since this is a right triangle, we can use trig functions

tan theta = opp / adj

tan 71 = 36/x

x tan 71 = 36

x = 36/tan 71

x=12.39579

To the nearest tenth

x = 12.4

An exam starts at 10:10 and lasts for 75 minutes what time does the exam end pls help i will give brainliest

Answers

Given:

An exam starts at 10:10 and lasts for 75 minutes.

To find:

The time when the exam end.

Solution:

We have,

Starting time = 10:10

Exam duration = 75 minutes

First we will divide the 75 minutes in two parts 50 minutes and 25 minutes.

We know that 50 minutes after 10:10 is 11:00 and 25 minutes after 11:00 is 11:25. So,

10:10 + 75 minutes = 10:10 + 50 minutes +25 minutes)

                               = (10:10 + 50 minutes) +25 minutes

                               = 11:00 +25 minutes

                               = 11:25

Therefore, the exam ends at 11:25.

Ilhan needs to write in function notation and evaluate this equation at the given value of the
independent variable. What answer should she get? 6x + y = 3; x=3

Answers

Answer:

it should be the second one I hope this help

Each side of a pentagon is 10 cm greater than the previous side. If the perimeter of this pentagon is 500 cm, find the lengths of the sides.

Answers

Answer: See explanation

Step-by-step explanation:

The perimeter of a pentagon is gotten through the summation of its five sides. Let the first side be represented by x. Since each side of a pentagon is 10 cm greater than the previous side, then the sides will be:

First side = x

Second side = x + 10

Third side = x + 10 + 10 = x + 20

Forth side = x + 30

Fifty side = x + 40

Therefore,

x + (x + 10) + (x + 20) + (x + 30) + (x + 40) = 500

5x + 100 = 500

5x = 500 - 100

5x = 400

x = 400/5

x = 80

Therefore, the lengths will be:

First side = x = 80cm

Second side = x + 10 = 80 + 10 = 90cm

Third side = x + 20 = 80 + 20 = 100cm

Forth side = x + 30 = 80 + 30 = 110cm

Fifty side = x + 40 = 80 + 40 = 120cm

Margaret took a trip to Italy. She had to convert US dollars to euros to pay for her expenses there. At the time she was traveling, the conversion rate was represented by the function , where n is the number of dollars and E(n) is the equivalent value in euros. Later, she traveled to Dubai and converted her remaining euros into the local currency, UAE dirhams. At that time, the conversion rate was represented by the function , where x is the number of euros and D(x) is the equivalent value in dirhams. Which function can be used to convert n dollars directly to dirhams?

Answers

The conversion rate US dollars to Euros is represented with the function:

E(n)=0.72n

n- number of dollars

E(n) - Euros as a function of US dollars

The conversion rate Euros to Dirhams is :

D(x)=5.10x

x- number of Euros

D(x)- Dirhams as a function of Euros

We are trying to find D(x) in terms of n.

D(x) = 5.10x

x can be rewritten as E(n)

D(x) = 5.10(E(n))

D(x) = 5.10(E(n))

D(x) = 5.10(0.72n)

D(x) = 3.672n

According to this the following statement is true:

A) (D x E)(n) = 5.10(0.72n)

Solve the following inequality: |x + 1| <_3

<_ = greater than or equal to

Answers

Answer:

x <= -4 or x >= 2

Step-by-step explanation:

so, it actually says

|x+1| >= 3

so, then, this is valid for all x >= 2 (then x+1 is 3 or higher), and for all x <= -4 (then x+1 is -3 or lower, and |x+1| is still 3 or higher)

12. (05.02 LC) Look at the figure below: an image of a right triangle is shown with an angle labeled y If sin y° = 7 over q and tan y° = 7 over r, what is the value of sec y°? (4 points) sec y° = q over r sec y° = 7r sec y° = 7q sec y° = r over q

Answers

Answer:

[tex]sec~y=\frac{q}{r}[/tex]

Step-by-step explanation:

[tex]tan ~y=\frac{7}{r} \\\\\frac{sin~y}{cos~y} =\frac{7}{r} \\\\sin~y=\frac{7}{r} cos~y\\sin ~y=\frac{7}{q} \\\frac{7}{q} =\frac{7}{r} cos~y\\sec~y=\frac{7}{r} \times \frac{q}{7} =\frac{q}{r}[/tex]

The tangent is equal to the product of the sine and secant. The value of sec y is q over r, then the correct option is A.

What is a right-angle triangle?

It is a type of triangle in which one angle is 90 degrees and it follows the Pythagoras theorem and we can use the trigonometry function. The Pythagoras is the sum of the square of two sides is equal to the square of the longest side.

If sin y = 7/q and tan y = 7/r.

Then the value of sec y will be

We know that the identity

[tex]\sec x =\dfrac{\tan x}{\sin x}[/tex]

Then we have

[tex]\rm \sec y = \dfrac{7/r}{7/q} \\\\\\\sec y = \dfrac{q}{r}[/tex]

The value of sec y is q over r.

More about the right-angle triangle link is given below.

https://brainly.com/question/3770177

please help with these two questions!!

Answers

6√5 + 3√6 = 6√5 + 3√6 [cannot be simplified]

; roots do not contain any perfect squares, and the roots are not similar.

6√5(3√6) = 18√30 [can be simplified]

; although roots do not contain any perfect squares, the product rule can be applied to create a singular expression.

Solve the inequality and write the solution in interval notation:
x-6/x+5 <0

(-5, 6)

[-5, 6)

(-infinity,-5) U [6,infinity)

(-infinity,-5] U (6,infinity)

Answers

Answer:

A

Step-by-step explanation:

Firstly x cannot be -5 because the expression on th left would be undefined so it's only between choices a and c.

Create a number line with makes the expression on left 0 and undefined...so at 6 and -5 this happens.

-------(-5)--------(6)---------

Let's test the 3 intervals by choosing a value from that interval to see if all numbers from that interval will make the expression on left less than 0.

Number before -5 is -6:

(-6-6)/(-6+5)=-12/-1=12 >0 so this interval is not a part of our solution.

Number between -5 and 6 is 0:

(0-6)/(0+5)=-6/5<0 so this interval is a part of our solution

Number after 6 like 7:

(7-6)/(7+5)=1/12>0 so this interval is not a part of our solution.

The winner is everything between-5 and 6 so answer is A.

Solve each of the following:
a) x² + 4x – 77 = 0
b) x(x + 4) = -2(3x + 8)
Please show your work

Answers

Answer:

a.) x=7 or x=-11

b.) x=−2 or x=−8

Step-by-step explanation:

a)  x² + 4x – 77 = 0

Step 1: Factor left side of equation.

(x−7)(x+11)=0

Step 2: Set factors equal to 0.

x−7=0 or x+11=0

x=7 or x=−11

b.) x(x + 4) = -2(3x + 8)

Step 1: Simplify both sides of the equation.

x^2+4x=−6x−16

Step 2: Subtract -6x-16 from both sides.

x^2+4x−(−6x−16)=−6x−16−(−6x−16)

x^2+10x+16=0

Step 3: Factor left side of equation.

(x+2)(x+8)=0

Step 4: Set factors equal to 0.

x+2=0 or x+8=0

x=−2 or x=−8

Answer:

a) {-11, 7}.

b) {-8, -2}

Step-by-step explanation:

a) x^2 + 4x - 77 = 0

To factor this we need 2 numbers whose product is -77 and sum is + 4.

They are + 11 and - 7, so:

( x + 11)(x - 7) = 0

x + 11 = 0 or x - 7 = 0

x = -11,  7.

b) x(x + 4) = -2(3x + 8)

x^2 + 4x = -6x - 16

x^2 + 4x + 6x + 16 = 0

x^2 + 10x + 16 = 0

(x + 2)(x + 8) = 0

x = -8, -2.

plzzz helppp only a hour due today

Answers

Answer:

A or C

Is my best I got stuck A or C

Answer:

[tex]\text{C. about }72.05\:\mathrm{cm^2}[/tex]

Step-by-step explanation:

This is a very fun problem that requires the use of multiple concepts to solve.

Concepts/formulas used:

The measure of an inscribed angle is half the measure of the arc it formsThere are 360 degrees in a circleThe sum of the interior angles of a triangle add up to 180 degreesLaw of Sines is given by [tex]\frac{\sin A}{a}=\frac{\sin B}{b}=\frac{\sin C}{c}[/tex]  All radii of a circle are exactly half all diameters of the circleThe area of a circle with radius [tex]r[/tex] is given by [tex]A=r^2\pi[/tex]

The measure of an inscribed angle is equal to half the measure of the arc it forms. In circle Z, [tex]\angle XVY[/tex] is an inscribed angle that forms arc XY. Since XY is 40 degrees, angle XVY must be [tex]40\div 2=20^{\circ}[/tex].

Similarly, [tex]\angle VYX[/tex] is also an inscribed angle and forms arc XV. Notice how arc XY and arc XV form arc VY, which is half the circumference of the circle, since segment VY is a diameter of the circle. Since there are 360 degrees in a circle, arc VY must be 180 degrees. Therefore, we have:

[tex]\widehat{XY}+\widehat{XV}=180^{\circ},\\\widehat{XV}+40^{\circ}=180^{\circ},\\\widehat{XV}=140^{\circ}[/tex]

Now we can find the measure of angle VYX, using our knowledge that the measure of an inscribed angle is half the measure of the arc it forms.

[tex]m\angle VYX=\frac{140}{2}=70^{\circ}[/tex]

Now, we have two angles of triangle VXY. Since the sum of the interior angles of a triangle add up to 180 degrees, the third angle, [tex]\angle VXY[/tex], can be found:

[tex]\angle VXY+\angle VYX+\angle XVY=180^{\circ},\\\angle VXY+20^{\circ}+70^{\circ}=180^{\circ},\\\angle VXY+90^{\circ}=180^{\circ},\\\angle VXY=90^{\circ}[/tex]

We can now use this angle and the Law of Sines to find the length of segment VY. The Law of Sines works for any triangle and is given by [tex]\frac{\sin A}{a}=\frac{\sin B}{b}=\frac{\sin C}{c}[/tex] (the ratio of any angle and its opposite side is maintained throughout all angles of the triangle).

Since angle VXY's opposite side is VY and angle VYX's opposite side is VX, we have the following proportion:

[tex]\frac{\sin 70^{\circ}}{9}=\frac{\sin 90^{\circ}}{VY}[/tex]

Recall that [tex]\sin 90^{\circ}=1[/tex]. Cross-multiply:

[tex]9\sin 90^{\circ}=VY\sin 70^{\circ},\\9=VY\sin 70^{\circ},\\VY=\frac{9}{\sin 70^{\circ}}[/tex]

This is the diameter of the circle. By definition, all radii are half the diameter. Therefore, the radius of the circle is [tex]\frac{9}{\sin 70^{\circ}}\cdot \frac{1}{2}=\frac{9}{2\sin 70^{\circ}}[/tex].

The area of a circle with radius [tex]r[/tex] is given by [tex]A=r^2\pi[/tex]. Substitute [tex]r=\frac{9}{2\sin 70^{\circ}}[/tex] to get the area of circle Z:

[tex]A=(\frac{9}{2\sin 70^{\circ}})^2\pi,\\A\approx (4.78879997614)^2\pi,\\A\approx 22.9326052115\pi,\\A\approx \boxed{72.05\:\mathrm{cm^2}}[/tex]

Can you answer this math homework? Please!

Answers

Answer:

Height is equal = Y = 1.8 X + 3.1 = 2.3 X + 1.9

=> 2.3 X - 1.8 X = 3.1 - 1.9

=> 0.5 X = 1.2

=> x = 1.2/0.5 = 2.4

Time = 2.4 weeks

Step-by-step explanation:

Answer:

2.4

Step-by-step explanation:

Multiply the binomials:

(y+2)•(y+9)

Answers

Answer:

y² + 11y + 18

Step-by-step explanation:

y² + 9y + 2y + 18

y² + 11y + 18

y^2+11y+18
explanation: y times y is y^2 y times 9 is 9y plus 2y then 2 times 9 which is 18

Solve the inequality.
-14> x-32
A. x>18
B. x< 46
C. x<-46
D. x < 18

Answers

Answer:

Option D

Step-by-step explanation:

-14 > x - 32

Add 32 to both sides;

18 > x OR x < 18

HELP?p?P?p?p?p?P?P?p?p?p?p?P?p?p?p?p?p?p?pp?p?p?P

Answers

Answer:

Yes, its a rational number.

Step-by-step explanation:

Rational numbers can be whole numbers, fractions, and decimals, and in this case it is a decimal.

Hope this helped!

Answer: yes

Step-by-step explanation:

yes 1.86 is a rational number

Solve for x. X/5-x/6=1/3 x = 10 x = 1/90 x = 1/10

Answers

Answer:

x=10

Step-by-step explanation:

I hope this will help you

10. Two planes are flying one directly behind the other. Both planes are at an alttude of 1.7 miles. The angle
of depression to the airport from the plane closer to the airport is 58. The angle of depression to the
airport from the plane farther from the airport is 37. What is the distance between the two planes to the
nearest tenth of a mile?
A 1.0
B 23 -
C 12
D Not here

Answers

It’s B trust me I have done this before

Find the values of the variables and please give the reasons​

Answers

180 ÷3 = 60 ° (angles on a straight line equals to 180 ° )

a+b+60 =180°

a+ b= 180 ° - 60° = 120°

(a+b) are 2 variables

so 120 ÷2 =60 °

therefore a and b =60 °

c +b+ 60° = 180° ( co - interior angles are supplementary angles )

c+60° +60° =180°

c +120° =180°

c =180°-120°

c=60 °

d=60° (alternate angles are equal )

or

c+b+d=180°

60° +60° + d = 180°

d=180°-120°

d=60°

(⅔)-⁴ (two over three to the power minus 4)​

I need answer asap pleaseeeee

Answers

Answer:

81/16

Step-by-step explanation:

(⅔)-⁴

81/16

= 5.0625

What is the value of y?

Answers

Answer:

C, 40 degrees

Step-by-step explanation:

All the angles of a triangle add to 180 degrees according to the Triangle Sum Theorem.

Since all angles sum to 180, we can set all the values to add to 180.

We have:

[tex]2y+y+10+50=180[/tex]

Combining like terms, we have:

[tex]3y+60=180[/tex]

Subtracting 60 from both sides gets us

[tex]3y=120[/tex]

Dividing by 3 from both sides equals

[tex]y=40[/tex]

Answer:

I think the value of y is 40

Step-by-step explanation:

Here, 2y+ y+ 10+50=180°( sum of all angles of triangle)or, 3y+ 60=180or, 3y=180-60or, 3y=120or, y= 120÷3:.y= 40

Prachi was 555 kilometers east of her home when she began driving farther east at 707070 kilometers per hour. Let f(n)f(n)f, (, n, )be Prachi's distance from her home at the beginning of the n^\text{th}n th n, start superscript, start text, t, h, end text, end superscript hour of her drive. fff is a sequence. What kind of sequence is it

Answers

Answer:

The answer is "[tex]\bold{f(n) = 70n + 5}[/tex]"

Step-by-step explanation:

The complete question is defined in the attached file Please find it.  

She started driving further east, 70 kilometers an hour, 5 km east of her home. Allow f(n) at the outset of its nth hour drive to just be Prachi's length from home.

F is a series of arithmetic.

Construct the series with an explicit formula.

[tex]\bold{f(n) = 70n + 5}[/tex]

Answer:

Arithmetic and it is f(n)=5+70(n-1)

Step-by-step explanation:

Khan Academy

What is the initial value of 34.2 x 3^x

Answers

Initial value is your y intercept, and to find that you just need to substitute 0 for x. Anything to the power of 0 is just 1. So you get 34.2(1), which means that your initial value is 34.2.

Determine the constant of variation for the direct variation given. (0, 0), (3, 12), (9, 36)
12
4
3

Answers

Answer:

4

Step-by-step explanation:

y = kx

Use point (3, 12).

12 = k * 3

k = 12/3 = 4

y = 4x

Answer: 4

Divide y by x:

12/3 = 4

36 / 9 = 4

The constant of variation is 4

Please help ASAP!!!!

Answers

Answers:  (4, 2) and (8, 2)

========================================================

Explanation:

The two points mentioned in bold are midpoints of segments AB and AC respectively.

To find the coordinates of a midpoint, you add up the x coordinates and divide by 2. Do the same with the y coordinates.

For example, points A and B are at (7,6) and (1,-2)

If we add up the x coordinates and divide by 2, then we get (7+1)/2 = 4. Do the same for the y coordinates to get (6+(-2))/2 = 2. So that's how (4,2) is the midpoint of segment AB. You'll use similar logic to find that (8,2) is the midpoint of segment AC.

A slight alternative is that once you find one midpoint is (4,2), you can draw a horizontal line until you reach (8,2). We're using the idea that the midsegment is parallel to BC which is also horizontal.

Sally is serving lemonade to four friends. She is serving 4/7 cup per person.

Estimate how much lemonade she needs. Then calculate exactly how much she needs. What is the difference between the estimate and actual amount?

pls help, :) ​

Answers

Answer:

oi ngl levi is hawt I like your pfp ^^

Step-by-step explanation:

my name is Riley

A student bought 84 pencils. If he sharpened 35 pencils, what is the ratio of the unsharpened pencils to the sharpened pencils?

Answers

Hello!

Sharpened => 35

Unsharpened => 84-35 = 49

49:35= 7:5

Good studies!

Answer:

7: 5

Step-by-step explanation:

unsharpened to sharpened

First we need to determine the number of unsharpened

84 - 35 =49

There are 35 sharpened

49:35

Divide each by 7

49/7 : 35/7

7: 5

What is the interquartile range of the following data set? 78,90,456,676,111,381,21

Answers

Answer

The IQR of the data set is 368.

Explanation

To find the interquartile range, you first need to find the median of the data set. Then, you find the median of the median and subtract them. This might be a little confusing but I'll walk through everything.

First, put the data set in order from least to greatest; 21 78 90 111 381 456 676. Find the median. The median of this data set is 111, since it is the middle number when the data set is ordered from least to greatest.

To find the Q1 and Q3 of the set, you have to find the median of the median.

The set right now is 21 78 90 111 381 456 676. Remove the 111 (if there were an even amount of numbers in the set, you wouldn't remove the 111 and you would just split the data set in half). Now you have two sets: 21 78 90 and 381 456 676. The median of the first set is 78 (this is the Q1) and the median of the second set is 456 (this is the Q3).

To find the interquartile range, subtract the Q1 from the Q3. 456-78=368.

Other Questions
In the function, g(x) = -2x , the independent variable has a value of 6. Find the value of the dependent variable. Find the surface area of the composite figure. Round to the nearest square centimeter Describe the transformation of f(x) to g(x). Pleaseee helllp thank youuuu!!! Least common factor how to do in 121,99 On a coordinate plane, a triangle has points A (negative 2, negative 2), B (1, negative 5), and C (negative 5, negative 5).If a translation of T2, 7(x, y) is applied to ABC, what are the coordinates of B'? Assume the existence of a Building class. Define a derived class, ApartmentBuilding that contains four (4) data members: an integer named numFloors, an integer named unitsPerFloor, a boolean named hasElevator, and a boolean named hasCentralAir. There is a constructor containing parameters for the initialization of the above variables (in the same order as they appear above). There are also two function: the first, getTotalUnits, accepts no parameters and returns the total number of units in the building; the second, isLuxuryBuilding accepts no parameters and returns true if the building has central air, an elevator and 2 or less units per floor. Empareja cada oracin con el futuro simple del verbo que se expresa con ir + infinitivo.Voy a ver si esa tienda vende prendas par uniformes. Despues hacer mi peinado voy a hacer el maquillaje. Si termino mis tareas voy a tener tiempo para jugar. Voy a ponerme una diadema. tendrharpodrvendrverpondr A 1030 kg car has four 12.0 kg wheels. When the car is moving, what fraction of the total kinetic energy of the car is due to rotation of the wheels about their axles Beta Company reported the following operating data for its baking division for the year: Sales $560,000 Contribution margin 308,000 Direct fixed costs 47,000 Average operating assets 230,000 How much is controllable margin for the year HI FRENCH QUESTION PLEASE! What might happen to the genes that break down carbohydrates when cats eat lipids and fats, but no carbohydrates? you are in the middle of discussing a patient cheif complaint,and reviewing the patient history with new patient history with a new pateint who seems to be little uncomfortable talking with you ,when another medical assistantknocks on the door and tells you that you have a call .what do you think that you should do andwhy? Carry out a tri-hybrid cross between XX(tall) YY(fat) ZZ(fair) and xx(short) yy(slim) zz(black). Find their genotypic and phenotypic ratios. Hey I need help I need to find English statements that are meaningful for example The sky is blue what are the reasons behind the different rituals of different caste/ethnic groups? Which is the best revision of this sentence?In a large box, the baker donated bread to the fund-raiser.The baker donated to the fund-raiser bread in a large box.The baker donated a large box of bread to the fund-raiser.In a large box, the fund-raiser had bread donated by the baker. Raymond Cattell's Personality Factor Questionnaire can be used toA. predict an individual's behavior.B. define a person's heredity.C. determine how stable a person is.D. measure a person's behavior. 14. The following solution contains errors. Identify the errors and explain why they are incorrect.Explain what should have been done to answer the question properly. a triangle has sides of 6 m 8 m and 11 m is it a right-angled triangle? Question 3 (Multiple Choice Worth 2 points)(01.03 MC)Why is it difficult to get people to follow regulations protecting a common?O People care more about money than the welfare of others.O Breaking these regulations does not come with direct penalties.The rules are too strict and prevent meeting basic needs.O Common area resources belong to everyone.